Rational sequence converging to irrational

In summary, the conversation discusses the lemma that for any irrational number there exists a sequence of rational numbers that converges to it, and how to proceed for a rigorous proof. The textbook uses the fact that ##\mathbb{Q}## is dense in ##\mathbb{R}##, but the conversation suggests using a topological approach to define the sequence. The idea is to define the sequence based on the decimal (or any other base) representation of the irrational number, and then use the fact that ##\mathbb{Q}## is dense in ##\mathbb{R}## to prove convergence. However, a more rigorous proof can be achieved by using the cauchy construction of ##\mathbb{
  • #1
Delta2
Insights Author
Gold Member
6,002
2,625
In the textbook I have (its a textbook for calculus from my undergrad studies, written by Greek authors) some times it uses the lemma that

"for any irrational number there exists a sequence of rational numbers that converges to it",

and it doesn't have a proof for it, just saying that it is a consequence of the fact that ##\mathbb{Q}## is dense in ##\mathbb R##.

Any ideas how to proceed for a rigorous proof?

My idea is that if ##x=x_0.x_1x_2...x_n,...## is the representation of the irrational x in the decimal system with ##x_i \in {0...9}## then the sequence

##\sigma_n=\sum\limits_{k=0}^{n}\frac{x_k}{10^k}## is rational and converges to the number but something tells me this is not a rigorous proof.
 
Last edited:
Mathematics news on Phys.org
  • #2
Delta² said:
In the textbook I have (its a textbook for calculus from my undergrad studies, written by Greek authors) some times it uses the lemma that

"for any irrational number there exists a sequence of rational numbers that converges to it",

and it doesn't have a proof for it, just saying that it is a consequence of the fact that ##\mathbb{Q}## is dense in ##\mathbb R##.

Any ideas how to proceed for a rigorous proof?
What does it mean, that ##\mathbb{Q}## is dense in ##\mathbb{R}##?
 
  • #3
That for any ##a,b## reals there exists ##c\in \mathbb Q## such that ##a<c<b##.
 
  • #4
Can you use this to find a sequence of rational numbers that converge to ##a##, if you keep ##a## fixed here?

Side remark: you should require a<b in the definition, otherwise a=2, b=1 is a counterexample.
 
  • #5
Delta² said:
That for any ##a,b## reals there exists ##c\in \mathbb Q## such that ##a<c<b##.
It might be easier to use the topological approach: given ##a \in \mathbb{R}## then every open neighborhood of ##a## contains a number ##c \in \mathbb{Q}##. Now all is left, is to define the sequence ##(c_n)_{n \in \mathbb{N}}##.
 
  • #6
That's where all my problem is to define the sequence, you got to give me as hint something more than the definition of dense. It seems there are infinitely many such sequences, got to find a way to choose one...
 
  • #7
You certainly have a lot of choice. Just pick an element, then think about how to find another element that is closer.
 
  • #8
Delta² said:
That's where all my problem is to define the sequence, you got to give me as hint something more than the definition of dense. It seems there are infinitely many such sequences, got to find a way to choose one...
Take open balls as neighborhoods and narrow down the diameter. The difficulty here is, that a hint and a solution is the same thing.
 
  • #9
sorry can't come up with any ideas (regarding the topological approach i was really bad at Topology) other than the one based in the decimal(or any other base) representation .

##x_0=[x]##

##x_n=\frac{[10^nx]}{10^n}## where [x] denotes the integer part of x.
 
  • #10
I think we can prove kind of easily that ##\lim x_n=x## (because ##[10^nx]=10^nx-\epsilon_n(x)## with ##0<\epsilon_n(x)<1## so that ##\frac{\epsilon_n(x)}{10^n}## converges to zero easily...

BUT the million dollar question,

"where do I need that ##\mathbb Q## is dense in ##\mathbb R##""?

(ok probably we need it for the rigorous definition of function [x]=the unique integer z such that z<x<z+1)
 
Last edited:
  • #11
O.k. let's take your definition. We have two real numbers ##a## and ##b=a+\dfrac{1}{n}## for ##n \in \mathbb{N}##.
Now you said, there is a rational number ##c_n## with ##a < c_n < a+\dfrac{1}{n}##.

So does the sequence ##(c_n)_{n \in \mathbb{N}}## converge, and if, what is the limit?
 
  • Like
Likes Delta2
  • #12
I wonder why I didn't think of that, I was trying to uniquely determine ##c_n## maybe that's why.
 
  • #13
If you want to learn more about it, you can read about the cauchy construction of ##\mathbb{R}##. Let me give you a brief sketch. Let's assume ##\mathbb{Q}## is given. I.e., we know such an ordered field exists.

Define the set ##F := \{(u_n)_n \in \mathbb{Q}^{\mathbb{N}} \mid \forall d \in \mathbb{Q^{+}: \exists n_0: \forall p,q > n_0: |u_p - u_q| < d}\}##

and the set ##N := \{(u_n)_n \in F \mid \forall d \in \mathbb{Q}^{+}: \exists n_0: \forall n > n_0: |a_n| < d\}##

While this may seem abstract, you can think about ##F## as the cauchy sequences and ##N## as the sequences converging to ##0##.

Now, you can define the equivalence relation ##\approx## on ##F##

such that ##(u_n) \approx (v_n) : \iff (u_n-v_n)_n \in N##

We then define ##\mathbb{R} := F/\approx##.

This means, we define the real numbers as the equivalence classes of sequences whose difference 'converges' to 0.

So in this context, real numbers are sets! One can prove (using denseness), that if we take any element out of this set (and such a set is never empty because equivalence classes are never empty, by reflexivity), that this sequence of rationals will converge to the real number it represents, as desired.
 
  • Like
Likes Delta2
  • #14
Math_QED said:
So in this context, real numbers are sets! One can prove (using denseness), that if we take any element out of this set (and such a set is never empty because equivalence classes are never empty, by reflexivity), that this sequence of rationals will converge to the real number it represents, as desired.
I would rather call it using ##\mathbb{R}## is the completion of ##\mathbb{Q}## by definition, and thus the result follows immediately from the convergence of Cauchy sequences. That was my first thought, too, but density is an unnecessary step here.
 

What is a rational sequence converging to an irrational number?

A rational sequence is a sequence of numbers that can be expressed as a ratio of two integers. An irrational number is a number that cannot be expressed as a ratio of two integers. A rational sequence converging to an irrational number is a sequence of rational numbers that gets closer and closer to an irrational number, but never reaches it.

Why is it important to study rational sequences converging to irrational numbers?

Studying rational sequences converging to irrational numbers can help us better understand the concept of irrational numbers and their properties. It can also aid in the development of computational methods for approximating irrational numbers and in solving mathematical problems involving irrational numbers.

How do you prove that a rational sequence converges to an irrational number?

To prove that a rational sequence converges to an irrational number, we need to show that the sequence gets arbitrarily close to the irrational number. This can be done by using the definition of convergence and the properties of irrational numbers.

What are some examples of rational sequences converging to irrational numbers?

One example is the sequence of rational numbers 1, 1.4, 1.41, 1.414, 1.4142, ... which converges to the irrational number √2. Another example is the sequence of rational numbers 3, 3.1, 3.14, 3.141, 3.1415, ... which converges to the irrational number π.

Are there any practical applications of rational sequences converging to irrational numbers?

Yes, there are many practical applications of rational sequences converging to irrational numbers in fields such as engineering, physics, and computer science. For example, in computer algorithms, irrational numbers are often approximated using rational sequences for more efficient and accurate calculations.

Similar threads

Replies
7
Views
1K
  • Calculus and Beyond Homework Help
Replies
13
Views
966
  • General Math
Replies
21
Views
3K
  • General Math
Replies
1
Views
2K
  • Math Proof Training and Practice
Replies
4
Views
1K
  • Topology and Analysis
Replies
3
Views
1K
Replies
7
Views
2K
  • Calculus and Beyond Homework Help
Replies
3
Views
1K
Replies
7
Views
1K
  • Topology and Analysis
2
Replies
44
Views
5K
Back
Top